he function f has a continuous derivative. if f(0)=1, f(2)=5, and ∫20f(x)ⅆx=7, what is ∫20x⋅f′(x)ⅆx ? (A) (B) (C) 10 (D) 17

Answers

Answer 1

The integration  ∫20x⋅f′(x)ⅆx is 1. The answer is (A) 1.

We can use integration by parts to solve this problem. Let u = x and v = f(x), then we have:

∫2^0 x f'(x) dx = [x f(x)]2^0 - ∫2^0 f(x) dx

Using the given values of f(0) and f(2), we get:

∫2^0 x f'(x) dx = -2f(0) + 2f(2) - ∫2^0 f(x) dx

Now, we need to find the value of ∫2^0 f(x) dx. We are given that ∫2^0 f(x) dx = 7, so substituting this value in the above equation, we get:

∫2^0 x f'(x) dx = -2 + 2f(2) - 7 = -9 + 2f(2)

We are also given that f(2) = 5, so substituting this value, we get:

∫2^0 x f'(x) dx = -9 + 2(5) = 1

Therefore, the answer is (A) 1.

To learn more about integration :

https://brainly.com/question/30215870

#SPJ11

Answer 2

We can solve this problem using integration by parts. Let's let u = x and dv = f'(x)dx, which means that du = dx and v = ∫f'(x)dx = f(x). Using the integration by parts formula, we get:

∫2 0 x*f'(x)dx = [x*f(x)]2 0 - ∫2 0 f(x)dx

We know that f(0) = 1 and f(2) = 5, so:

[x*f(x)]2 0 = 2*5 - 0*1 = 10

Now we need to evaluate ∫2 0 f(x)dx. We know that ∫2 0 f(x)dx = 7, so:

∫2 0 x*f'(x)dx = 10 - 7 = 3

Therefore, the answer is (B) 3.
To find the value of the integral ∫2₀xf′(x)dx, we can use integration by parts. Let u = x and dv = f′(x)dx. Then, du = dx and v = ∫f′(x)dx = f(x).

Now apply the integration by parts formula: ∫udv = uv - ∫vdu. So, ∫2₀xf′(x)dx = xf(x)│₂₀ - ∫2₀f(x)dx.

Evaluate the terms: (2f(2) - 0f(0)) - ∫2₀f(x)dx = (2 * 5) - (0 * 1) - 7 = 10 - 7 = 3.

Therefore, the value of the integral ∫2₀xf′(x)dx is 3, which corresponds to option (B).

To learn more about Integration: brainly.com/question/30900582

#SPJ11


Related Questions

what’s this ? i need the answer because i need some better understanding

Answers

The equivalent expression of  (r/s)(6) is determined as (3 (6) - 1 ) / ( 2(6) + 1).

Option A.

What is the equivalent expression?

The equivalent expression that represents (r/s)(6) is calculated by substituting the given values of r and s as follows;

The given expression;

r = 3x - 1

s = 2x + 1

Now, we are going to find the value of the expression [r/s] (6) as follows;

( 3x - 1 ) / (2x + 1) ( 6 )

Simplify further and we will have;

So we will replace, x with 6, to obtain the desired expression;

(3 (6) - 1 ) / ( 2(6) + 1)

This expression corresponds to the solution in option A.

Thus, the equivalent expression of  (r/s)(6) is determined as (3 (6) - 1 ) / ( 2(6) + 1) as shown in option A.

Learn more about equivalent expression here: https://brainly.com/question/22365614

#SPJ1

(1 point) consider the following two systems. (a) {−3x−2y2x−3y==−2−2 (b) {−3x−2y2x−3y==2−4 (i) find the inverse of the (common) coefficient matrix of the two systems.

Answers

The inverse of the coefficient matrix is [tex]\left[\begin{array}{cc}\frac {-3}{13}&\frac {2}{13}\\\frac {2}{13}&\frac {-3}{13}\end{array}\right][/tex]

How to find the inverse of the common coefficient matrix?

To find the inverse of the common coefficient matrix of the two systems, we first need to write the matrix in question.

We can do this by taking the coefficients of the variables and arranging them in a matrix.

For the systems (a) and (b), the coefficient matrices are:

A =[tex]\left[\begin{array}{cc}-3&-2\\2&-3\end{array}\right][/tex]

To find the inverse of matrix A, we can use the formula:

[tex]A^-1 = (1/det(A)) * adj(A)[/tex]

where det(A) is the determinant of A and adj(A) is the adjugate (or classical adjoint) of A.

First, let's find the determinant of A:

det(A) = (-3)(-3) - (2)(-2) = 9 - (-4) = 13

Next, we need to find the adjugate of A. To do this, we need to find the transpose of the matrix of cofactors of A. The matrix of cofactors of A is:

C =[tex]\left[\begin{array}{cc}-3&-2\\2&-3\end{array}\right][/tex]

Note that the cofactor of aij is [tex](-1)^{(i+j)}[/tex] times the determinant of the matrix obtained by deleting row i and column j of A. Using this rule, we can find the matrix of cofactors C.

C =[tex]\left[\begin{array}{cc}-3&2\\2&-3\end{array}\right][/tex]

Now we need to find the transpose of C, which is:

[tex]C^T[/tex] =[tex]\left[\begin{array}{cc}-3&2\\2&-3\end{array}\right][/tex]

Finally, we can find the inverse of A using the formula:

[tex]A^-1 = (1/det(A)) * adj(A)[/tex]

[tex]A^-1 = (1/13) *\left[\begin{array}{cc}-3&2\\2&-3\end{array}\right][/tex]

[tex]A^-1 =\left[\begin{array}{cc}\frac {-3}{13}&\frac {2}{13}\\\frac {2}{13}&\frac {-3}{13}\end{array}\right][/tex]

Therefore, the inverse of the common coefficient matrix of the two systems is:

[tex]\left[\begin{array}{cc}\frac {-3}{13}&\frac {2}{13}\\\frac {2}{13}&\frac {-3}{13}\end{array}\right][/tex]

Learn more about inverse of coefficient matrix

brainly.com/question/29131275

#SPJ11

Naomi plotted the graph below to show the relationship between the temperature of her city and the number of popsicles she sold daily:



Part A: In your own words, describe the relationship between the temperature of the city and the number of popsicles sold. (2 points)



Part B: Describe how you can make the line of best fit. Write the approximate slope and y-intercept of the line of best fit. Show your work, including the points that you use to calculate the slope and y-intercept. (3 points)

Answers

Part A: The relationship between the temperature of Naomi’s city and the number of popsicles she sold daily is direct and proportional. This implies that as the temperature of the city increases, the number of popsicles sold per day also increases. This is confirmed by the upward trend of the graph, which shows an increase in the number of popsicles sold per day as the temperature increases.

Part B: The line of best fit is a straight line that is used to represent the trend of a scatter plot. The line of best fit can be used to make predictions about the value of the dependent variable based on the value of the independent variable. To create the line of best fit for this graph, we need to identify the slope and y-intercept.

The slope of the line of best fit can be calculated using the formula:

slope = (y2 - y1)/(x2 - x1)
where (x1, y1) and (x2, y2) are any two points on the line of best fit. We can choose two points on the line of best fit, such as (20, 25) and (40, 75), and substitute the values into the formula:
slope = (75 - 25)/(40 - 20)
slope = 50/20
slope = 2.5
The approximate slope of the line of best fit is 2.5.
The y-intercept of the line of best fit can be calculated by substituting the slope and one of the points on the line of best fit into the formula:
y - y1 = m(x - x1)

where m is the slope and (x1, y1) is one of the points on the line of best fit. We can choose the point (20, 25) and substitute the values into the formula:
y - 25 = 2.5(x - 20)
y - 25 = 2.5x - 50
y = 2.5x - 25
The y-intercept of the line of best fit is -25.
Therefore, the line of best fit for the graph is:
y = 2.5x - 25.

To know more about temperature visit:

brainly.com/question/15267055

#SPJ11

Suppose f is increasing on the interval [a, b] and we want to estimate the area under the curve on this interval. 1. If f is concave down on this interval, using left endpoints would give

Answers

If f is increasing on the interval [a, b] and concave down, using left endpoints to estimate the area under the curve would give an overestimate of the actual area.

To see why, consider dividing the interval [a, b] into n subintervals of equal width Δx = (b-a)/n. Let x0 = a, x1 = a + Δx, x2 = a + 2Δx, ..., xn = b be the endpoints of these subintervals. Then, the left endpoints approximation to the area under the curve is given by the Riemann sum:

R_n = Δx[f(x0) + f(x1) + f(x2) + ... + f(x(n-1))]

Since f is increasing, f(x0) ≤ f(x1) ≤ f(x2) ≤ ... ≤ f(x(n-1)) ≤ f(xn). Since f is concave down, its graph is below any secant line connecting two of its points. Therefore, the Riemann sum using left endpoints overestimates the area of the region under the curve, because the rectangles defined by the left endpoints have height f(x0), f(x1), ..., f(x(n-1)) and their top sides are above the curve.In contrast, using right endpoints to estimate the area would give an underestimate, because the rectangles would have their bottom sides above the curve.

Therefore, the best approximation using rectangles would be the midpoint Riemann sum, which uses the midpoint of each subinterval as the height of the rectangle. This approximation is always between the left and right endpoint approximations and is closer to the actual area under the curve.

To learn more about “rectangles” refer to the https://brainly.com/question/25292087

#SPJ11

if tan ( x ) = 5 9 (in quadrant-i), find cos ( 2 x ) =

Answers

The Pythagorean identity if tan ( x ) = 5 9 (in quadrant-i),  cos(2x) = 56/53.

If tan(x) = 5/9 in quadrant I, we can use the Pythagorean identity to find cos(x):

cos(x) = 1/sqrt(1 + tan^2(x)) = 9/√(5^2 + 9^2) = 9/√106.

To find cos(2x), we can use the double angle formula for cosine:

cos(2x) = 2cos^2(x) - 1 = 2(9/√106)^2 - 1 = (162/106) - 1 = 56/53.

Therefore, cos(2x) = 56/53.

Learn more about Pythagorean identity here

https://brainly.com/question/24287773

#SPJ11

Find the sum of the series sigma^infinity_n = 0 (-1)^n 3^nx^2n/n! sigma^infinity_n = 0 3^n+1x^2n/n!

Answers

To find the sum of the series sigma^infinity_n = 0 (-1)^n 3^nx^2n/n! and sigma^infinity_n = 0 3^n+1x^2n/n!, we can use the formula for the sum of an infinite geometric series:

S = a / (1 - r)

where S is the sum, a is the first term, and r is the common ratio.

For the first series, a = 1 and r = -3x^2 / (n+1)(n+2). To see this, note that the nth term of the series is (-1)^n 3^n x^2n / n!, and the ratio between consecutive terms is -3x^2 / (n+1)(n+2). Therefore, the sum of the series is:

S = 1 / (1 + 3x^2/2 + 9x^4/8 + ...)

For the second series, a = 3x^2 and r = 3x^2 / (n+2)(n+3). To see this, note that the nth term of the series is 3^(n+1) x^2n / (n+1)!, and the ratio between consecutive terms is 3x^2 / (n+2)(n+3). Therefore, the sum of the series is:

S = 3x^2 / (1 - 3x^2/6 + 9x^4/120 - ...)

Both of these series converge for all values of x, so the sums exist. However, neither series has a closed-form expression in terms of elementary functions, so the above expressions are the best we can do.

Learn more about geometric series here:

https://brainly.com/question/4617980

#SPJ11

Functions x(t) and h(t) have the waveforms shown in Fig. 2.12. Determine and plot y(t) = x(t) * h(t) using the following methods. (a) Integrating the convolution analytically. (b) Integrating the convolution graphically. 2.12 Functions x(t) and ht) have the waveforms shown in Fig.P2.12. Determine and plot yt=xt*h(t using the following methods (a) Integrating the convolution analytically (b) Integrating the convolution graphically x(t) h(t) 2 0 0 0 t(s) 0 LS 1 1 2 Figure P2.12:Waveforms for Problem 2.12

Answers

y(t) = 2t^2 - 12t + 16 for 0 ≤ t ≤ 2, and y(t) = 0 otherwise, using both methods of integrating the convolution.

To determine and plot y(t) = x(t) * h(t), where * represents convolution, using the given waveforms, we can use two methods: (a) integrating the convolution analytically and (b) integrating the convolution graphically.

(a) Integrating the convolution analytically:

The convolution of two functions f(t) and g(t) is given by the integral of the product of the two functions over all possible values of the variable t:

f(t) * g(t) = ∫ f(τ)g(t-τ) dτ

where τ is a dummy variable of integration.

Using this formula, we can compute y(t) = x(t) * h(t) as follows:

y(t) = ∫ x(τ)h(t-τ) dτ

= ∫ x(τ)h(2-t-τ) dτ (since h(t) is non-zero only for 0 ≤ t ≤ 2)

= ∫ x(τ)h(2-t)h(τ-t+2) dτ (using the time reversal property of h(t))

= h(2-t) ∫ x(τ)h(τ-t+2) dτ (since h(2-t) is constant w.r.t τ)

= 2(2-t) ∫ 2(τ-t+2) dτ (since x(t) is constant w.r.t τ and h(τ-t+2) is zero outside the interval [t-2, t])

= (2-t) [τ^2-2tτ+8τ] from τ=0 to τ=2-t

= 2t^2 - 12t + 16 for 0 ≤ t ≤ 2

= 0 otherwise

(b) Integrating the convolution graphically:

To integrate the convolution graphically, we can plot x(t) and h(t) on the same graph and slide h(t) along the t-axis, multiplying it with x(t) at each value of t and adding up the products to obtain y(t).

From the given waveforms, we can plot x(t) and h(t) on the same graph as follows:

x(t) is a rectangular pulse of width 1 and amplitude 2, centered at t=0.5.

h(t) is a triangular pulse of base width 2 and peak amplitude 1, centered at t=1.

Now, we slide h(t) along the t-axis and multiply it with x(t) at each value of t as shown in the attached image. At t=0, h(t) and x(t) do not overlap, so their product is zero.

At t=1, h(t) and x(t) overlap partially, so we multiply x(t) with the overlapping part of h(t) and obtain a trapezoidal pulse of amplitude 2.

At t=2, h(t) and x(t) overlap completely, so we multiply x(t) with h(t) and obtain a triangular pulse of amplitude 2.

Adding up the products at each value of t, we obtain y(t) as shown in the attached image. The resulting waveform is a piecewise linear function of t, with maximum amplitude 4 and zero outside the interval [0, 2].

In summary, we have obtained the same result, y(t) = 2t^2 - 12t + 16 for 0 ≤ t ≤ 2, and y(t) = 0 otherwise, using both methods of integrating the convolution.

To know more about convolution refer here :

https://brainly.com/question/31656685#

#SPJ11

Consider the vector field F (x, y, z) = (5z + 4y) i + (2z + 4x) j + (2y + 5x) k. Find a function f such that F = nabla f and/(0, 0, 0) = 0. f(x, y, z) = Suppose C is any curve from (0, 0, 0) to (1, 1, 1). Use part a

Answers

To find a function f such that F = ∇f and f(0, 0, 0) = 0, we need to determine the potential function associated with the vector field F. The function f(x, y, z) = 2xy + 2xz + 2yz satisfies the conditions and is the desired potential function.

In order for a vector field F to have a potential function, it must satisfy the condition ∇ × F = 0, where ∇ is the gradient operator. Computing the curl of the given vector field F (5z + 4y)i + (2z + 4x)j + (2y + 5x)k, we find that ∇ × F = 0, indicating that F has a potential function.

To find the potential function f(x, y, z), we integrate each component of F with respect to its corresponding variable. Integrating the x-component gives 2xy + g(y, z), integrating the y-component gives 2xz + g(x, z), and integrating the z-component gives 2yz + g(x, y). Here, g(y, z), g(x, z), and g(x, y) represent arbitrary functions of their respective variables.

Since the gradient of a scalar function is unique up to an additive constant, we can choose g(y, z), g(x, z), and g(x, y) to be zero. Therefore, the potential function f(x, y, z) = 2xy + 2xz + 2yz satisfies F = ∇f, and f(0, 0, 0) = 0 as desired.

For any curve C from (0, 0, 0) to (1, 1, 1), we can calculate the line integral of F along C by evaluating f at the endpoints and subtracting the values. Using f(1, 1, 1) - f(0, 0, 0), we obtain the desired result.

Learn more about gradient here:

https://brainly.com/question/30249498

#SPJ11

The graphs below show the test scores for students in different subject areas and the time the students spent studying
for the tests.
Math Scores vs. Hours Spent Studying
100 x
8882889822
Test Score
90-
80
70
60
50
40
30
20-
10-
0.5 1 1.5 2 2.5 3 3.5 4 4.5
Hours Spent Studying
Spelling Scores vs. Hours Spent Studying
Science Scores vs. Hours Spent Studying
100 I
90
Test Score
888888888
80
70
60
50
40
30
20-
10
0.5 1 1.5 2 2.5 3 3.5 4 4.5
Hours Spent Studying
History Scores vs. Hours Spent Studying
Save and Exit
Next
Submit

Answers

Answer:

The area of one side of a cuboid is 360cm. What is the length, if the width is 1.5cm?

4.2. use the fourier transform analysis equation (4.9) to calculate the fourier transforms of: (a) b(t 1) b(t- 1) (b) fr{u( -2- t) u(t- 2)}

Answers

(a) the Fourier transform of b(t+1) b(t-1) is the square of the Fourier transform of b(t).

(a) Let's use the Fourier transform analysis equation (4.9) to find the Fourier transform of b(t+1) b(t-1):

F{b(t+1) b(t-1)} = ∫₋∞^∞ b(t+1) b(t-1) e₋ⱼωt dt

Let's make a substitution to simplify the expression:

u = t + 1, du = dt

v = t - 1, dv = dt

t = (u + v) / 2

dt = (du + dv) / 2

Substituting, we get:

F{b(t+1) b(t-1)} = ∫₋∞^∞ b(u) b(v) e₋ⱼω[(u+v)/2] (du+dv)/2

= 1/2 ∫₋∞^∞ [b(u) e₋ⱼωu] [b(v) e₋ⱼωv] e₋ⱼωu/2 e₋ⱼωv/2 du dv

= 1/2 ∫₋∞^∞ [b(u) e₋ⱼωu/2] [b(v) e₋ⱼωv/2] e₋ⱼω(u+v)/2 du dv

= 1/2 ∫₋∞^∞ [b(u) e₋ⱼωu/2] e₋ⱼωu/2 du ∫₋∞^∞ [b(v) e₋ⱼωv/2] e₋ⱼωv/2 dv

= [F{b(t)}]²

(b) Let's use the Fourier transform analysis equation (4.9) to find the Fourier transform of u(-2-t) u(t-2):

F{u(-2-t) u(t-2)} = ∫₋∞^∞ u(-2-t) u(t-2) e₋ⱼωt dt

Note that u(-2-t) is equal to 1 for t ≤ -2 and 0 otherwise, while u(t-2) is equal to 1 for t ≥ 2 and 0 otherwise. Therefore, the product u(-2-t) u(t-2) is equal to 1 for t between -2 and 2, and 0 otherwise. Using this information, we can write:

F{u(-2-t) u(t-2)} = ∫₋₂^₂ e₋ⱼωt dt

Integrating, we get:

F{u(-2-t) u(t-2)} = [e₋ⱼωt / ⱼω]₋₂^₂ = [e₋ⱼ2ω - e₋ⱼ(-2ω)] / ⱼω

Simplifying, we get:

F{u(-2-t) u(t-2)} = (sin(2ω) / ω) e₋ⱼω

Therefore, the Fourier transform of u(-2-t) u(t-2) is (sin(2ω) / ω) e₋ⱼω.

To learn more about Fourier transform visit:

brainly.com/question/29063535

#SPJ11




Please I need help with this ASAP.



A teacher is playing a game with her students. She prepared 23 cards. Each card has a number from 1 to 23. She has 30 students in her class. She will pick 4 students from the class and ask them to draw 4 cards. Each student will pick one card only. These 4 numbers will create a secret code to a locker.


a) What is the probability that the secret code is composed of numbers with GCD 4?


b) If her top four students picked the numbers, what is the probability of getting at least 3 prime numbers?

Answers

The probability that the secret code is composed of numbers with a GCD of 4 is approximately 0.68%.

a) The probability that the secret code is composed of numbers with a greatest common divisor (GCD) of 4 can be determined by finding the total number of favorable outcomes and dividing it by the total number of possible outcomes.

To have a GCD of 4, the numbers must be divisible by 4. Out of the 23 available cards, there are 5 numbers (4, 8, 12, 16, and 20) that are divisible by 4.

Since each student picks one card, the first student has a 5/23 chance of selecting a card divisible by 4. Once the first card is selected, there are 4/22 cards remaining for the second student, 3/21 for the third student, and 2/20 for the fourth student.

To calculate the overall probability, we multiply the probabilities of each student's selection:

P(GCD 4) = (5/23) * (4/22) * (3/21) * (2/20) ≈ 0.0068 or 0.68%

Therefore, the probability that the secret code is composed of numbers with a GCD of 4 is approximately 0.68%.

b) If the top four students picked the numbers, we need to determine the probability of getting at least 3 prime numbers.

There are 9 prime numbers between 1 and 23 (2, 3, 5, 7, 11, 13, 17, 19, 23). We will calculate the probability of picking 3 prime numbers and 4 prime numbers separately, and then add them together.

P(3 prime numbers) = (9/23) * (8/22) * (7/21) * (14/20)

P(4 prime numbers) = (9/23) * (8/22) * (7/21) * (6/20)

To find the probability of getting at least 3 prime numbers, we add these two probabilities:

P(at least 3 prime numbers) = P(3 prime numbers) + P(4 prime numbers)

The result will give us the probability of obtaining at least 3 prime numbers when the top four students pick the numbers.

Learn more about probability here:

https://brainly.com/question/32117953

#SPJ11

if you have 18 dimes and
Quaters that are worth
2.25, which system would
represent this

Answers

The correct expression is,

⇒ $1.8 + 0.25y = $2.25

Where, y is number of quarters.

We have to given that;

You have 18 dimes and Quarters that are worth $2.25.

Since, We know that;

1 dimes = 0.10 dollar

1 quarters = 0.25 dollar

Hence, We get;

18 dimes = 18 x 0.10

              = 1.8 dollars

So, We can formulate the correct expression which represent the situation is,

⇒ $1.8 + 0.25y = $2.25

Where, y is number of quarters.

Learn more about the measurement unit visit:

https://brainly.com/question/777464

#SPJ1

evaluate the integral. 4 1 t3 t2 − 4 dt 2√2

Answers

The given integral is evaluated using integration by substitution method. Let u = t2 – 4, then du/dt = 2t. Rewriting the integral in terms of u gives ∫(4/(2√2)) (u+4)^(3/2) du. Now applying the power rule of integration, we get (4/(5√2)) (u+4)^(5/2) + C. Substituting back u = t2 – 4, we get the final result as (4/(5√2)) (t2)^(5/2) – (4/(5√2)) (2^(5/2)) + C.

The given integral can be written as ∫(4/(2√2)) (t3/(t2 – 4)) dt. To evaluate this integral, we use integration by substitution method. Let u = t2 – 4, then du/dt = 2t. Solving for dt, we get dt = du/(2t). Substituting these values in the integral, we get ∫(4/(2√2)) ((t2 – 4 + 4)/(t2 – 4))^(3/2) (du/(2t)). Simplifying this, we get ∫(4/(2√2)) ((u+4)/(u))^(3/2) (du/(4√2)). Cancelling the 4s and 2s, we get ∫(u+4)^(3/2)/(u^(1/2)) du.
Now, using the power rule of integration, we get (4/(5√2)) (u+4)^(5/2) + C. Substituting back u = t2 – 4, we get the final result as (4/(5√2)) (t2)^(5/2) – (4/(5√2)) (2^(5/2)) + C.

The given integral is evaluated using integration by substitution method. The substitution u = t2 – 4 is used to simplify the integral. The final result is obtained by substituting the value of u back in the expression.

To know more about substitution method visit:

https://brainly.com/question/30284922

#SPJ11

Use Green's Theorem to evaluate the line integral along the given positively oriented curve.
?C 4sin(y)dx + 4xcos(y)dy
C is the ellipse x2 + xy + y2 = 25

Answers

The line integral is zero: ∫C 4sin(y)dx + 4xcos(y)dy = 0.

To apply Green's Theorem, we need to find the curl of the vector field F = (4sin(y), 4xcos(y)). We have:

∂F2/∂x = 4cos(y)

∂F1/∂y = 4cos(y)

So the curl of F is:

curl(F) = ∂F2/∂x - ∂F1/∂y = 0

Since the curl of F is zero, we can apply Green's Theorem to find the line integral along the ellipse C:

∫C F · dr = ∬R curl(F) dA = 0

where R is the region enclosed by C, and dA is an infinitesimal area element.

Therefore, the line integral is zero:

∫C 4sin(y)dx + 4xcos(y)dy = 0

So the answer is 0.

Learn more about integral here:

https://brainly.com/question/18125359

#SPJ11:

A company sells widgets. The amount of profit, y, made by the company, is related to the selling price of each widget, x, by the given equation. Using this equation, find out what price the widgets should be sold for, to the nearest cent, for the company to make the maximum profit. Y=−44x2+1375x−6548y=-44x^2+1375x-6548y=−44x2+1375x−6548

Answers

To determine the price of widgets that a company should sell to maximize profit, you need to find the value of x at which the given equation will produce the highest y value.

Here's how to solve this:

Step 1: Rewrite the equation in standard form y = -44x² + 1375x - 6548 becomes

y = -44(x² - 31.25x) - 6548

Step 2: Complete the square by adding and subtracting the square of half of the coefficient of x:

y = -44(x² - 31.25x + (31.25/2)² - (31.25/2)²) - 6548

y = -44((x - 15.625)² - 244.141) - 6548

y = -44(x - 15.625)² + 10723.564

Step 3: The maximum value of y occurs when

(x - 15.625)² = 244.141/44.

Therefore,

x - 15.625 = ±sqrt(244.141/44)

x = 15.625 ± 2.765

x = 18.39 or 12.86

Since the company cannot sell at a negative price, x must be $12.86 or $18.39.

The company should sell widgets at $12.86 or $18.39 to maximize profit to the nearest cent.

To know more about price visit:

https://brainly.com/question/19091385

#SPJ11

Find formulas for the entries of A^t, where t is a positive integer. Also, find the vector A^t [1 3 4 3]

Answers

The entries of A^t, where t is a positive integer. The values of P and simplifying, we get A^t [1 3 4 3] = [(1/3)(-1 + 3t), (1/3)(2 + t), (1/3)(-1 + 2t)].

Let A be an n x n matrix and let A^t denote its t-th power, where t is a positive integer. We can find formulas for the entries of A^t using the following approach:

Diagonalize A into the form A = PDP^(-1), where D is a diagonal matrix with the eigenvalues of A on the diagonal and P is the matrix of eigenvectors of A.

Then A^t = (PDP^(-1))^t = PD^tP^(-1), since P and P^(-1) cancel out in the product.

Finally, we can compute the entries of A^t by raising the diagonal entries of D to the power t, i.e., the (i,j)-th entry of A^t is given by (D^t)_(i,j).

To find the vector A^t [1 3 4 3], we can use the formula A^t = PD^tP^(-1) and multiply it by the given vector [1 3 4 3] using matrix multiplication. That is, we have:

A^t [1 3 4 3] = PD^tP^(-1) [1 3 4 3] = P[D^t [1 3 4 3]].

To compute D^t [1 3 4 3], we first diagonalize A and find:

A = [[1, -1, 0], [1, 1, -1], [0, 1, 1]]

P = [[-1, 0, 1], [1, 1, 1], [1, -1, 1]]

P^(-1) = (1/3)[[-1, 2, -1], [-1, 1, 2], [2, 1, 1]]

D = [[1, 0, 0], [0, 1, 0], [0, 0, 2]]

Then, we have:

D^t [1 3 4 3] = [1^t, 0, 0][1, 3, 4, 3]^T = [1, 3, 4, 3]^T.

Substituting this into the equation above, we obtain:

A^t [1 3 4 3] = P[D^t [1 3 4 3]] = P[1, 3, 4, 3]^T.

Using the values of P and simplifying, we get:

A^t [1 3 4 3] = [(1/3)(-1 + 3t), (1/3)(2 + t), (1/3)(-1 + 2t)].

Learn more about positive integer here

https://brainly.com/question/16952898

#SPJ11

for what values of x does the graph of f (x) = ex −2x have a horizontal tangent line?

Answers

The graph of the function f(x) = ex - 2x has a horizontal tangent line at x = 0.693.

To find the values of x for which the graph of the function f(x) = ex - 2x has a horizontal tangent line, we need to determine when the derivative of the function is equal to zero. A horizontal tangent line occurs when the slope of the function is zero, which corresponds to the critical points of the function.

To find the critical points, we differentiate f(x) with respect to x. The derivative of ex is ex, and the derivative of -2x is -2. Setting the derivative equal to zero, we have ex - 2 = 0.

Adding 2 to both sides, we get ex = 2. Taking the natural logarithm of both sides, we have ln(ex) = ln(2), which simplifies to x = ln(2).

Therefore, the graph of f(x) = ex - 2x has a horizontal tangent line at x = ln(2) or approximately x = 0.693. At this point, the slope of the function is zero, indicating a horizontal tangent line.

Learn more about tangent line here:

https://brainly.com/question/31617205

#SPJ11

If 43% of American pet owners keep a photograph of their pet in their wallet, find the probability that 5 randomly selected American pet owners will have a photograph of their pet in their wallet. Please round the final answer to 2 or 3 decimal places

Answers

The probability of a randomly selected American pet owner keeping a photograph of their pet in their wallet is 43% or 0.43.

To find the probability that 5 randomly selected American pet owners will have a photograph of their pet in their wallet, we use the binomial probability formula:

[tex]P(X = k) = C(n, k) * p^k * (1 - p)^(n - k)[/tex]

where:

P(X = k) is the probability of exactly k successes,

C(n, k) is the number of combinations of n items taken k at a time,

p is the probability of success for one trial,

n is the total number of trials.

In this case, k = 5, p = 0.43, and n = 5.

Plugging in the values, we get:

[tex]P(X = 5) = C(5, 5) * 0.43^5 * (1 - 0.43)^(5 - 5)[/tex]

[tex]P(X = 5) = 1 * 0.43^5 * (1 - 0.43)^0[/tex]

[tex]P(X = 5) = 0.43^5[/tex]

Calculating this probability, we get:

P(X = 5) ≈ 0.0439

Rounded to 2 decimal places, the probability that 5 randomly selected American pet owners will have a photograph of their pet in their wallet is approximately 0.04.

Learn more about probability here:

https://brainly.com/question/30853716

#SPJ11

Given g(x)=x11−3x9+2, find the x-coordinates of all local minima using the second derivative test. If there are multiple values, give them separated by commas. If there are no local minima, enter ∅.

Answers

The x-coordinates of all local minima using the second derivative test is [tex](27/11)^(^1^/^2^).[/tex]

First, we need to find the critical points by setting the first derivative equal to zero:

g'(x) = [tex]11x^10 - 27x^8[/tex] = 0

Factor out x^8 to get:

[tex]x^8(11x^2 - 27)[/tex] = 0

So the critical points are at x = 0 and x =  ±[tex](27/11)^(^1^/^2^).[/tex]

Next, we need to use the second derivative test to determine which critical points correspond to local minima. The second derivative of g(x) is:

g''(x) =[tex]110x^9 - 216x^7[/tex]

Plugging in x = 0 gives g''(0) = 0, so we cannot use the second derivative test at that critical point.

For x = [tex](27/11)^(^1^/^2^)[/tex], we have g''(x) = [tex]110x^9 - 216x^7 > 0[/tex], so g(x) has a local minimum at x =[tex](27/11)^(^1^/^2^).[/tex]

For x = -[tex](27/11)^(^1^/^2^)[/tex], we have g''(x) = [tex]-110x^9 - 216x^7 < 0[/tex], so g(x) has a local maximum at x = -[tex](27/11)^(^1^/^2^)[/tex]

Therefore, the x-coordinates of the local minima of g(x) are [tex](27/11)^(^1^/^2^).[/tex]

Know more about derivative here:

https://brainly.com/question/23819325

#SPJ11

how many 5-digit numbers are there in which every two neighbouring digits differ by ?

Answers

There are no 5-digit numbers in which every two neighboring digits differ by 2.

This is because if we start with an even digit in the units place, the next digit must be an odd digit, and then the next digit must be an even digit again, and so on. However, there are no pairs of adjacent odd digits that differ by 2.

Similarly, if we start with an odd digit in the units place, the next digit must be an even digit, and then the next digit must be an odd digit again, and so on. But again, there are no pairs of adjacent even digits that differ by 2.

Therefore, there are 0 5-digit numbers in which every two neighboring digits differ by 2.

Learn more about neighboring here

https://brainly.com/question/23792839

#SPJ11

Basketball player Chauncey Billups of the Detroit pistons makes free throw shots 88% of the time. Find the probability that he misses his first shot and makes the second. a 0.5000 b 0,7744 c 0.1056 d 0.0144

Answers

The probability that Chauncey Billups misses his first free throw and makes the second is 0.1056. This probability is obtained by multiplying the probability of missing a free throw (0.12) with the probability of making a free throw (0.88). Answer is c) 0.1056.

To calculate the probability, we first determine that the probability of missing a free throw is 1 - 0.88 = 0.12, as Billups makes free throws 88% of the time.The probability that Chauncey Billups misses his first free throw and makes the second can be calculated by multiplying the probabilities of each event.

Given that he makes free throw shots 88% of the time, the probability of missing a free throw is 1 - 0.88 = 0.12.

To find the probability of missing the first shot and making the second, we multiply the probabilities: 0.12 * 0.88 = 0.1056.

Therefore, the correct answer is c) 0.1056.

Learn more about probability : brainly.com/question/31828911

#SPJ11

The new circular community swimming pool has a diameter of 64 feet

Answers

A circular swimming pool with a diameter of 64 feet would have a radius of 32 feet. This means that the distance from the center of the pool to any point on the edge (or circumference) would be 32 feet.

The area of a circle can be calculated using the formula A = πr²,

where A represents the area and r represents the radius. In this case, the radius is 32 feet, so the area of the pool would be:

A = π × (32 feet)²

A = π × 1024 square feet

A ≈ 3.14 × 1024 square feet

A ≈ 3,210.24 square feet

So, the approximate area of the circular community swimming pool would be around 3,210.24 square feet.

To learn more about area of a circle visit:

brainly.com/question/28642423

#SPJ11

The new circular community swimming pool has a diameter of 64 feet. A. What is the area of the community pool?

consider the hypothesis test: , vs where is the slope of the linear model relating y: son's height to x: father's height. what is the observed value of the test statistic for this hypothesis test?

Answers

Assuming you have the sample data available, here's the general procedure to calculate the observed value of the test statistic for a hypothesis test:

Collect a sample of father-son pairs and record their heights (x and y, respectively).

Calculate the correlation coefficient (r) between the father's height (x) and the son's height (y). This will give an estimate of the strength and direction of the linear relationship.

Compute the observed value of the test statistic using the formula:

t = (r * sqrt(n - 2)) / sqrt(1 - r^2),

where n is the sample size.

Note: The test statistic for testing the slope of a linear model is typically the t-statistic, which follows a t-distribution under the null hypothesis.

Once you have the observed value of the test statistic (t), you can compare it to the critical value(s) or calculate the p-value to make a conclusion about the hypothesis test.

Please provide the sample data if you have it, and I can assist you in calculating the observed value of the test statistic.

To determine the observed value of the test statistic for the hypothesis test comparing the slope of the linear model, we need some additional information. Specifically, we require the sample data consisting of pairs of father's height (x) and son's height (y).

Assuming we have the necessary data, we can proceed with the hypothesis test. The null hypothesis, denoted as H0, states that the slope of the linear model relating the son's height (y) to the father's height (x) is equal to zero. The alternative hypothesis, denoted as Ha, asserts that the slope is not equal to zero.

In hypothesis testing, the test statistic measures the difference between the observed data and what is expected under the null hypothesis. For a hypothesis test concerning the slope of a linear regression model, the appropriate test statistic is typically the t-statistic.

The formula for the t-statistic in this context is:

t = (b - 0) / se(b),

where b is the estimated slope coefficient from the linear regression model, and se(b) is the standard error of the slope coefficient.

By plugging in the observed values for the slope coefficient and the standard error, we can calculate the t-statistic. This t-statistic represents the observed value of the test statistic for the hypothesis test.

It's important to note that without the actual data and relevant statistical output, it is not possible to provide a specific numerical value for the observed test statistic. The calculation depends on the sample data and the estimation of the slope coefficient from the linear regression model.

Learn more about Hypothesis Testing :

https://brainly.com/question/32387002

#SPJ11

question 2 options: if random variable x has a binomial distribution with n=15 and p(success) =p= 0.6, find the mean of x. that is, find e(x). round to the whole number. do not use decimals. answer:

Answers

The mean of X, or the expected value of X, is 9. This means that if we were to conduct the same experiment numerous times, on average, we would expect to observe 9 successes per 15 trials.

In probability theory, a binomial distribution is a discrete probability distribution that describes the number of successes in a fixed number of independent trials with a constant probability of success. In this case, we have a random variable X that has a binomial distribution with parameters n = 15 and p = 0.6. We are required to find the mean of X, denoted as E(X).

The mean of a binomial distribution is given by the formula E(X) = np, where n is the number of trials and p is the probability of success in each trial. Substituting the given values, we get E(X) = 15 x 0.6 = 9.

It's worth noting that the mean of a binomial distribution represents a measure of central tendency and can be used to make predictions about the likely number of successes in future trials. Additionally, the variance and standard deviation of the binomial distribution can also be calculated using formulas, and these measures provide information about the spread or dispersion of the distribution.

For such more questions on Numerous times:

https://brainly.com/question/1323509

#SPJ11

The mean of X or the expected value of X is 9. This means that if we run the same test many times, on average, we expect to observe 9 successes each time experiment 15.

In probability theory, the binomial distribution is a probability variable that describes the number of successes of a fixed number of experiments. In this case, we have a random variable X that follows a binomial distribution with parameters n = 15 and p = 0.6.

We need to find the mean of X, the mean of E(X).

The mean of the binomial distribution is given by the formula E(X) = np; where n is the number of trials and p is the probability for each trial. Substituting the given values, we get E(X) = 15 x 0.6 = 9.

The binomial distribution represents a measure of central tendency and validity for predicting the number of future successes. trials.

In addition, the model can be used to calculate the variance and standard deviation of the binomial distribution, and these measures provide information about the distribution of the distribution.

Learn more about Mean:

brainly.com/question/31101410

#SPJ11

Evaluate the line integral sc F .dr, where C is given by the vector function r(t). 19. Flx, y) - xy'i - x'j.

Answers

Answer:

The value of the line integral s F .dr is -1/4 + 2/3j.

To evaluate the line integral s F .dr, where C is given by the vector function r(t) = ⟨x(t), y(t)⟩, we need to find the limits of integration and express F in terms of r(t).

First, let's find the limits of integration. We are not given any specific values of t, so we need to find the range of t that corresponds to the curve C. Since C is not explicitly defined, we can use the parameterization r(t) = ⟨t, t^2⟩ as a possible representation of C. We can see that as t varies, r(t) traces out a parabola in the xy-plane. Therefore, we can take the limits of integration to be the range of t that corresponds to this parabolic segment. One way to find this range is to solve the quadratic equation y = x^2 for x in terms of y, which gives x = ±√y. Since we are only interested in the part of the parabola that lies in the first quadrant, we take x = √y. Thus, the limits of integration are t = 0 to t = 1.

Next, let's express F in terms of r(t). We have F(x, y) = ⟨-xy, -x⟩ = -xy⟨1, 0⟩ - x⟨0, 1⟩ = -xyi - xj. To express F in terms of r(t), we substitute x = t and y = t^2, which gives F(r(t)) = -t^3i - tj.

Now we can evaluate the line integral using the formula

s F .dr = ∫a^b F(r(t)) . r'(t) dt,

where r'(t) = ⟨dx/dt, dy/dt⟩ is the derivative of r(t). In our case, r'(t) = ⟨1, 2t⟩.

Thus, we have

s F .dr = ∫0^1 F(r(t)) . r'(t) dt
= ∫0^1 (-t^3i - tj) . ⟨1, 2t⟩ dt
= ∫0^1 (-t^3 + 2t^2j) dt
= [-1/4t^4 + 2/3t^3j]0^1
= (-1/4 + 2/3j) - (0 + 0j)
= -1/4 + 2/3j.

Therefore, the value of the line integral s F .dr is -1/4 + 2/3j.

Know more about Integrals here:

https://brainly.com/question/18125359

#SPJ11

Carl wants to install new flowing in his hallway and kitchen. He does not need new flooring in the stove,counter, or sink areas. How many square feet of flooring will he need to purchase?



A:388ft


B:334ft


C:390ft


D:456ft

Answers

To determine the square footage of flooring needed, we need to calculate the total area of the hallway and kitchen, excluding the stove, counter, and sink areas.

Carl will need to purchase 388 square feet of flooring for his hallway and kitchen.

Let's assume the hallway and kitchen have rectangular shapes. We need to measure the length and width of each area and calculate their individual areas. Then, we can add the areas together to find the total square footage.

Once we have the measurements, we can sum up the area of the hallway and the kitchen while subtracting the area of the stove, counter, and sink areas.

After performing the calculations, we find that the total area of flooring needed is 388 square feet.

Therefore, Carl will need to purchase 388 square feet of flooring for his hallway and kitchen. The correct answer is A: 388ft.

Learn more about area here:

https://brainly.com/question/1631786

#SPJ11

90 points

Factor the following polynomial completely.

- x2y2 + x4 + 9 y2 - 9 x2

( x + 3)( x - 3)( x + y )( x - y )
( x - 3)( x - 3)( x + y )( x - y )
( x + 3)( x + 3)( x + y )( x - y )

Answers

Answer: A) (x + 3)(x - 3)(x + y)(x - y)

Step-by-step explanation:

The correct factorization of the polynomial -x^2y^2 + x^4 + 9y^2 - 9x^2 is:

(x + 3)(x - 3)(x + y)(x - y)

This factorization is obtained by grouping terms and factoring out common factors.

use binomial series to approximate 3√29 accurate to 0.0001. hint: let f(x) = 3√27 x = 3 ( 1 x 27 )1/3 , then find an approximation for f(2). hint: remember the alternating series estimate

Answers

An approximation of 3√29 accurate to 0.0001 is 3.1058 (rounded to four decimal places).

We can use the binomial series expansion to approximate the function f(x) = 3√x as follows:

f(x) = x^(1/3) = (1 + (x - 1))^(1/3)

Using the binomial series expansion for (1 + t)^n, where t = x - 1 and n = 1/3, we have:

f(x) = (1 + (x - 1))^(1/3) = 1 + (1/3)(x - 1) - (1/9)(x - 1)^2 + (4/81)(x - 1)^3 - (14/243)(x - 1)^4 + ...

Now, we can substitute x = 29 and truncate the series at the term involving (x - 1)^4, since we want an accuracy of 0.0001. We get:

f(29) ≈ 1 + (1/3)(28) - (1/9)(28)^2 + (4/81)(28)^3 - (14/243)(28)^4

f(29) ≈ 3.105835

Know more about binomial series here:

https://brainly.com/question/30177068

#SPJ11

what is the power of the eye in diopters when viewing an object 65 cm away

Answers

The power of the eye is, 51.54 diopters

Since, We know that;

The power of the eye is given by;

P = 1/f = 1/dₙ + 1/dₐ

where;

P is the power of the eye in diopter

f is the focal length of the eye

dₙ is the distance between the eye and the object

dₐ is the distance between the eye and the image

Given;

dₙ = 65 cm = 0.65 m

dₐ = 2.0 cm = 0.02 m

Hence,

P = 1/0.65 + 1/0.02

P = 1.54 + 50

P = 51.54 diopters

Therefore, the power of the eye is 51.54 diopters.

Learn more about the addition visit:

https://brainly.com/question/25421984

#SPJ1

suppose x has a continuous uniform distribution over the interval [1.7, 5.2]. round your answers to 3 decimal places. (a) determine the mean of x.

Answers

(a) The mean of x is 3.450

To determine the mean of x, where x has a continuous uniform distribution over the interval [1.7, 5.2], you need to follow these steps:

Step 1: Identify the lower limit (a) and upper limit (b) of the interval. In this case, a = 1.7 and b = 5.2.

Step 2: Calculate the mean (μ) using the formula: μ = (a + b) / 2.

Step 3: Plug in the values of a and b into the formula: μ = (1.7 + 5.2) / 2.

Step 4: Calculate the mean: μ = 6.9 / 2 = 3.45.

Therefore, the mean of x is 3.450 when rounded to 3 decimal places.

Know more about mean here:

https://brainly.com/question/1136789

#SPJ11

Other Questions
What is x - 12 = 6THANKSSSSSSSSS Stem-Changing ("Boot") Verbs/ Verbos con cambios radicalesThere are three main typesof stem-changing verbs in the present tense:andWhen we say verbs have "stem-changes we mean that part of the infinitivechanges. Thevowel in the infinitive changes in everyandforms.form of the verb BUT theStem-changing verbs are also known as __bost VectsPrctica: First, underline or circle which vowel in the following verbs changes. The stem changesare already given to you. Then, conjugate four of the verbs (one of each kind) in the space below.O>UEEIEEIPoderTo be able toQuerer = To wantServir = To servePreferir = To preferMedir = To measure Jugar" = To playPedir - to ask for!order Dormir = To sleepVenir" = To comeVerbo: Querer-Verbo: poderVerbo: pedir:Verbo: jugar lower cabinet is suitable for storing and stocking pots and pans true or false Find the measure of the indicated angle to the nearest degree.PLS URGENT HELP You deposit $400 Into a bank account. The account pays 2.75% annual interest compounded semi-annually. Find the balance after 5 years 95 divided by 9 please helppppppp why the normal respiration in adult (14-24) per minutes but in infant (24-34) per minutes? 84 miles in 12 days how many miles a day El rea cuadrada de un tablero de ajedrez mide 1681 cm.Cul es la medida de cada uno de sus lados? Please help me I will offer brainliest. Find |14.06|= I'm having trouble with this. James determined that these two expressions were equivalent expressions using the values of x = 4 and x = 6. Which statements are true? Check all that apply.7 x + 4 and 3 x + 5 + 4 x minus 1When x = 2, both expressions have a value of 18.The expressions are only equivalent for x = 4 and x = 6.The expressions are only equivalent when evaluated with even values.The expressions have equivalent values for any value of x.The expressions should have been evaluated with one odd value and one even value.When x = 0, the first expression has a value of 4 and the second expression has a value of 5.The expressions have equivalent values if x = 8. In a basketball game, Sally is the top scorer with 35% of the team'spoints. If the team scored 120 points in total, how many points did Sallyscore? The specific heat of gold is 0.126 J/gC. How much energy is releasedwhen a 350 g sample of gold is cooled from 88.5C to 24.6C? Which value of x makes the two matrices inverses of each other?521-2215-2-12 Sodium metal reacts with chlorine gas to produce crystals of sodiumchloride. Show phases in your equation. To rent a limousine, Theo must pay an initial fee of $45 in addition to $12 for every hour (h) he has the limousine reserved. To calculate the final cost (c), Theo wrote the following equation.c=45+12hIf the final bill amounts to $123, how many hours did Theo have the limousine reserved An equilateral triangle has a side length of 30.Another side of the same triangle measures 3x + y.The third side of the same triangle measures 4x - 2y.What are the values of x and y that would keep the triangle equilateral? Katie wants to collect over 100 seashells. She already has 34 seashells in her collection. Each day, she finds 12 more seashells on the beach. Katie can use fractions of days to find seashells. Write an inequality to determine the number of days, d, it will take Katie to collect over 100 seashells. what were the subjects or themes used